Diễn Đàn MathScopeDiễn Đàn MathScope
  Diễn Đàn MathScope
Ghi Danh Hỏi/Ðáp Thành Viên Social Groups Lịch Ðánh Dấu Ðã Ðọc

Go Back   Diễn Đàn MathScope > Sơ Cấp > Việt Nam và IMO > 2012

News & Announcements

Ngoài một số quy định đã được nêu trong phần Quy định của Ghi Danh , mọi người tranh thủ bỏ ra 5 phút để đọc thêm một số Quy định sau để khỏi bị treo nick ở MathScope nhé !

* Nội quy MathScope.Org

* Một số quy định chung !

* Quy định về việc viết bài trong diễn đàn MathScope

* Nếu bạn muốn gia nhập đội ngũ BQT thì vui lòng tham gia tại đây

* Những câu hỏi thường gặp

* Về việc viết bài trong Box Đại học và Sau đại học


Trả lời Gởi Ðề Tài Mới
 
Ðiều Chỉnh Xếp Bài
Old 11-01-2012, 11:43 AM   #1
n.v.thanh
Moderator
 
n.v.thanh's Avatar
 
Tham gia ngày: Nov 2009
Bài gởi: 2,849
Thanks: 2,980
Thanked 2,537 Times in 1,008 Posts
[VMO 2012] Bài 4 - Tổ Hợp

Bài 4 (5 điểm) .
Cho số nguyên dương $n $. Có $n $ học sinh nam và $n $ học sinh nữ xếp thành một hàng ngang, theo thứ tự tùy ý. Mỗi học sinh (trong số $2n $ học sinh vừa nêu) được cho một số kẹo bằng đúng số cách chọn ra hai học sinh khác giới với X và đứng ở hai phía của $X $. Chứng minh rằng tổng số kẹo mà tất cả $2n $ học sinh nhận được không vượt quá $\frac{1}{3}n(n^2-1) $.
[RIGHT][I][B]Nguồn: MathScope.ORG[/B][/I][/RIGHT]
 
n.v.thanh is offline   Trả Lời Với Trích Dẫn
The Following 4 Users Say Thank You to n.v.thanh For This Useful Post:
hoduckhanhgx (11-01-2012), Kém Toán (11-01-2012), nhox12764 (11-01-2012), trang96 (11-01-2012)
Old 11-01-2012, 11:52 AM   #2
shido_soichua
Maths is my life
 
shido_soichua's Avatar
 
Tham gia ngày: Oct 2009
Đến từ: Ninh Bình
Bài gởi: 300
Thanks: 31
Thanked 132 Times in 76 Posts
Gửi tin nhắn qua Yahoo chát tới shido_soichua
Trích:
Nguyên văn bởi n.v.thanh View Post
Bài 4 (5 điểm) .
Cho số nguyên dương $n $. Có $n $ học sinh nam và $n $ học sinh nữ xếp thành một hàng ngang, theo thứ tự tùy ý. Mỗi học sinh (trong số $2n $ học sinh vừa nêu) được cho một số kẹo bằng đúng số cách chọn ra hai học sinh khác giới với X và đứng ở hai phía của $X $. Chứng minh rằng tổng số kẹo mà tất cả $2n $ học sinh nhận được không vượt quá $\frac{1}{3}n(n^2-1) $.
Có ai làm bài này ko ?
Nếu các học sinh xếp xen kẽ thì số keeoj đúng bằng $\frac{1}{3}n(n^2-1) $
Nếu các học sinh không xếp xen kẽ xét 1 nhóm học sinh nam cạnh cùng với 2 học sinh nữ ở 2 đấu nhóm này.
Gọi k là số nữ ở bên trái nhóm nam
p là số nam ở bên trái bạn nữ đầu tiên bên trái nhóm nam này
p là số nam ở bên trái bạn nữ đầu tiên bên phải nhom nam này.
Khi đó ta đổi chô 2 bạn nam trên với 2 bạn nữ ở cạnh. Gọi $S_m $ là số kẹo sau m lần chuyển đổi. Khi đó ta có:
$S_{m+1}=S_m+2q-2p-4 $ mặt khác $q>p+2 $ do có ít nhất 2 nam ở giữa nên $S_{m+1}>S_m $
Do số kẹo nhận đc có giới hạn nên sau 1 số lần chuyển đổi quá trình trên dừng lại khi đó không còn 2 bạn nam đứng cạnh nhau. Ta có ĐPCM
[RIGHT][I][B]Nguồn: MathScope.ORG[/B][/I][/RIGHT]
 
__________________
http://luongvantuy.org/forum.php
Chuyên Văn - Lương Văn Tụy
shido_soichua is offline   Trả Lời Với Trích Dẫn
The Following 6 Users Say Thank You to shido_soichua For This Useful Post:
caubedien (11-01-2012), ghetvan (11-01-2012), Hoanglong2011 (11-01-2012), Kém Toán (11-01-2012), nhox12764 (11-01-2012), sine (11-01-2012)
Old 11-01-2012, 12:01 PM   #3
duynhan
+Thành Viên+
 
Tham gia ngày: Dec 2009
Bài gởi: 231
Thanks: 103
Thanked 118 Times in 68 Posts
Trích:
Nguyên văn bởi n.v.thanh View Post
Bài 4 (5 điểm) .
Cho số nguyên dương $n $. Có $n $ học sinh nam và $n $ học sinh nữ xếp thành một hàng ngang, theo thứ tự tùy ý. Mỗi học sinh (trong số $2n $ học sinh vừa nêu) được cho một số kẹo bằng đúng số cách chọn ra hai học sinh khác giới với X và đứng ở hai phía của $X $. Chứng minh rằng tổng số kẹo mà tất cả $2n $ học sinh nhận được không vượt quá $\frac{1}{3}n(n^2-1) $.
Với nam nữ xen kẽ ta có số kẹo đúng bằng $\frac13 n(n^2-1) $.
Xét 1 cách xếp mà nam nữ không xen kẽ, gọi i là vị trí nhỏ nhất sao cho từ 1,..,i-1 xếp xen kẽ, học sinh thứ i khác giới với học sinh thứ i-1 và cùng giới với i+1, giả sử có m bạn nữ xếp kề nhau $(m \ge 2) $, phía trước có x nam và y nữ, do xếp xen kẽ nên $y \le x \le y+1 $.
Tính được tổng số kẹo.
Đổi bạn nam vào vị trí i+1 ta cũng tính được số kẹo.
Xét hiệu tổng 2 - tổng 1 thì ta được >=0.
[RIGHT][I][B]Nguồn: MathScope.ORG[/B][/I][/RIGHT]
 
__________________

thay đổi nội dung bởi: duynhan, 11-01-2012 lúc 12:05 PM
duynhan is offline   Trả Lời Với Trích Dẫn
The Following User Says Thank You to duynhan For This Useful Post:
dzitxiem (12-01-2012)
Old 11-01-2012, 12:13 PM   #4
Hoanglong2011
+Thành Viên+
 
Tham gia ngày: Sep 2011
Bài gởi: 7
Thanks: 3
Thanked 0 Times in 0 Posts
Trích:
Nguyên văn bởi shido_soichua View Post
Có ai làm bài này ko ?
Nếu các học sinh xếp xen kẽ thì số keeoj đúng bằng $\frac{1}{3}n(n^2-1) $
Nếu các học sinh không xếp xen kẽ xét 1 nhóm học sinh nam cạnh cùng với 2 học sinh nữ ở 2 đấu nhóm này.
Gọi k là số nữ ở bên trái nhóm nam
p là số nam ở bên trái bạn nữ đầu tiên bên trái nhóm nam này
p là số nam ở bên trái bạn nữ đầu tiên bên phải nhom nam này.
Khi đó ta đổi chô 2 bạn nam trên với 2 bạn nữ ở cạnh. Gọi $S_m $ là số kẹo sau m lần chuyển đổi. Khi đó ta có:
$S_{m+1}=S_m+2q-2p-4 $ mặt khác $q>p+2 $ do có ít nhất 2 nam ở giữa nên $S_{m+1}>S_m $
Do số kẹo nhận đc có giới hạn nên sau 1 số lần chuyển đổi quá trình trên dừng lại khi đó không còn 2 bạn nam đứng cạnh nhau. Ta có ĐPCM
Mình làm tương tự vậy.
Quan trọng là 2 bước:
1. CM cách xếp xen kẽ thì số kẹo đúng bằng $\frac{n(n^2-1)}{3} $
2. CM với số HS là 2n, cách xếp trên có số kẹo lớn nhất. (bằng cách sử dụng đơn biến)

Nhưng mà cái bước 1 CM hơi ẩu (do ko có time).
Hy vọng đc 4 đ bài này
[RIGHT][I][B]Nguồn: MathScope.ORG[/B][/I][/RIGHT]
 

thay đổi nội dung bởi: Hoanglong2011, 11-01-2012 lúc 01:38 PM
Hoanglong2011 is offline   Trả Lời Với Trích Dẫn
Old 11-01-2012, 01:48 PM   #5
Brandnewworld
+Thành Viên+
 
Tham gia ngày: Dec 2009
Bài gởi: 25
Thanks: 10
Thanked 8 Times in 4 Posts
Trích:
Nguyên văn bởi Hoanglong2011 View Post
Mình làm tương tự vậy.
Quan trọng là 2 bước:
1. CM cách xếp xen kẽ thì số kẹo đúng bằng $\frac{n(n^2-1)}{3} $
2. CM với số HS là 2n, cách xếp trên có số kẹo lớn nhất. (bằng cách sử dụng đơn biến)

Nhưng mà cái bước 1 CM hơi ẩu (do ko có time).
Hy vọng đc 4 đ bài này
Bạn này có ý tưởng giống mình:
_B1: CM "thuật toán tối ưu" khi nam,nữ xếp xen kẽ nhau.
_B2: CM số kẹo vừa đúng bằng $1/3n(n^2-1) $

Nhưng tiếc là B1 mình làm ẩu hết 2 chỗ, không biết được mấy điểm nữa
[RIGHT][I][B]Nguồn: MathScope.ORG[/B][/I][/RIGHT]
 
Brandnewworld is offline   Trả Lời Với Trích Dẫn
Old 11-01-2012, 01:59 PM   #6
ThangToan
+Thành Viên+
 
Tham gia ngày: Nov 2010
Đến từ: THPT chuyên Vĩnh Phúc
Bài gởi: 570
Thanks: 24
Thanked 537 Times in 263 Posts
Trích:
Nguyên văn bởi n.v.thanh View Post
Bài 4 (5 điểm) .
Cho số nguyên dương $n $. Có $n $ học sinh nam và $n $ học sinh nữ xếp thành một hàng ngang, theo thứ tự tùy ý. Mỗi học sinh (trong số $2n $ học sinh vừa nêu) được cho một số kẹo bằng đúng số cách chọn ra hai học sinh khác giới với X và đứng ở hai phía của $X $. Chứng minh rằng tổng số kẹo mà tất cả $2n $ học sinh nhận được không vượt quá $\frac{1}{3}n(n^2-1) $.
Bài này có thể làm như sau:
Trước hết ta đánh số $2n $ học sinh có vị trí là $1, 2, ..., n $. Giả sử học sinh nam ở các vị trí $i_1, i_2, ..., i_n $. Khi đó với học sinh nam ở vị trí thứ $i_k $ thì số kẹo nhận được là: $\[\left( {{i_k} - k} \right)\left( {n + k - {i_k}} \right)\] $. Do đó tổng số kẹo n học sinh nam nhận được là:
$\[\sum\limits_{k = 1}^n {\left( {{i_k} - k} \right)\left( {n + k - {i_k}} \right)} \] $.
Tiếp theo ta tính số kẹo của học sinh nữ.
Số kẹo mà các học sinh nữ ở vị trí $<i_1 $ bằng 0. Số kẹo mà các học sinh nữ ở bị trí $>i_n $ bằng 0.
số kẹo mà các học sinh nữ ở vị trí h sao cho $i_k<h<i_{k+1}; k=1,...,n-1 $ bằng $\[k\left( {n - k} \right)\left( {{i_{k + 1}} - {i_k} - 1} \right)\] $ suy ra tổng số kẹo mà n học sinh nữ nhận được là:
$\[\sum\limits_{k = 1}^{n - 1} {k\left( {n - k} \right)\left( {{i_{k + 1}} - {i_k} - 1} \right)} $
Do đó tổng số kẹo các học sinh nhận được bằng:
$\sum\limits_{k = 1}^n {\left( {{i_k} - k} \right)\left( {n + k - {i_k}} \right)} +\sum\limits_{k = 1}^{n - 1} {k\left( {n - k} \right)\left( {{i_{k + 1}} - {i_k} - 1} \right)} $
Sau đó chứng minh
$\sum\limits_{k = 1}^n {\left( {{i_k} - k} \right)\left( {n + k - {i_k}} \right)} +\sum\limits_{k = 1}^{n - 1} {k\left( {n - k} \right)\left( {{i_{k + 1}} - {i_k} - 1} \right)}\le \frac{1}{3}n(n^2-1) $
Thật vậy, $\sum\limits_{k = 1}^n {\left( {{i_k} - k} \right)\left( {n + k - {i_k}} \right)} +\sum\limits_{k = 1}^{n - 1} {k\left( {n - k} \right)\left( {{i_{k + 1}} - {i_k} - 1} \right)} $
$= - \frac{{\sum\limits_{k = 1}^n {{{\left( {{i_k} - 2k} \right)}^2}} + \sum\limits_{k = 1}^n {{{\left( {{i_k} - 2k + 1} \right)}^2}} + n}}{2} + \frac{{n\left( {{n^2} - 1} \right)}}{3} $ (1)
Với chú ý ${\left( {{i_k} - 2k} \right)^2} + {\left( {{i_k} - 2k + 1} \right)^2} \ge 1, \forall k, 1\le k\le n $ nên từ (1) ta có đpcm.
[RIGHT][I][B]Nguồn: MathScope.ORG[/B][/I][/RIGHT]
 

thay đổi nội dung bởi: ThangToan, 12-01-2012 lúc 04:47 AM
ThangToan is offline   Trả Lời Với Trích Dẫn
Old 11-01-2012, 02:16 PM   #7
kirin
+Thành Viên+
 
Tham gia ngày: Apr 2011
Đến từ: http://m.facebook.com/story.php?story_fbid=488454984546725&id=165605226827592&refid=17&ref=stream
Bài gởi: 13
Thanks: 1
Thanked 0 Times in 0 Posts
Bài này mình chỉ làm đc bước tính số kẹo khi xếp xen kẽ còn b2 cm khá vớ vẩn gọi là nêu ra hướng giải là chính, không biết đc mấy điểm
[RIGHT][I][B]Nguồn: MathScope.ORG[/B][/I][/RIGHT]
 
kirin is offline   Trả Lời Với Trích Dẫn
Old 11-01-2012, 02:46 PM   #8
Mashimaru
+Thành Viên+
 
Tham gia ngày: Mar 2008
Bài gởi: 89
Thanks: 19
Thanked 70 Times in 28 Posts
Gọi $a_1, a_2, ..., a_n $ và $b_1, b_2, ..., b_n $ là vị trí của $n $ nam và $n $ nữ trên hàng. Xét nam tại vị trí $a_i $, ta thấy bên trái anh ta có $a_i - 1 $ vị trí, trong đó có $i - 1 $ vị trí là nam, vậy nên bên trái anh ta có $a_i - i $ nữ. Tương tự, bên phải anh ta có $n - (a_i - i) $ nữ. Vậy nam tại $a_i $ được cho $(a_i - i)(n - (a_i - i)) $ kẹo. Tương tự, nữ tại vị trí $b_i $ được cho $(b_i - i)(n - (b_i - i)) $ kẹo.

Tính tổng số kẹo được cho, chú ý $\{a_1, a_2, ..., a_n, b_1, b_2, ..., b_n\} = \{1, 2, ..., 2n\} $, ta đưa yêu cầu của bài toán thành việc chứng minh bất đẳng thức $S = \sum_{i = 1}^{n} i (a_i + b_i) \leq \frac{n(n + 1)(8n + 1)}{6} $.

Để ý rằng dấu bằng trong bất đẳng thức trên đạt được nếu $a_i = 2i - 1 $ và $b_i = 2i $, với mọi $i = 1, ..., n $ (hoặc ngược lại). Vậy ta sẽ xét cấu hình có $a_i = 2i - 1 $ và $b_i = 2i $, với mọi $i $, cùng với giả sử $a_1 < b_1 $. Từ cấu hình này, để có được một cấu hình bất kỳ với người đứng đầu hàng là nam (tức là $a_1 = 1 $), ta có thể thực hiện liên tiếp các phép đổi vị trí hai người liên tiếp trên hàng. Ta sẽ chứng minh phép đổi này không làm tăng tổng $S $.

Thật vậy, có hai khả năng:
1. Nếu ta đổi vị trí 2 nam hoặc 2 nữ liên tiếp thì dễ thấy tổng số kẹo không đổi.
2. Nếu ta đổi chỗ nam ở vị trí $a_i $ với nữ ở vị trí $b_j $ (chú ý rằng muốn làm được như vậy thì ta phải có $|a_i - b_j| $ = 1). Không mất tính tổng quát, giả sử $a_i < b_j $ thì $b_j = a_i + 1 $, sau khi đổi xong, ta được $a_i $ và $b_j + 1 $. Lúc này, tổng S nói chung không đổi, trừ phần $i(a_i + b_i) + j(a_j + a_i + 1) $ trở thành $i (a_i + 1) + j(a_j + a_i) $, do đó tổng GIẢM một lượng bằng $i - j $ (ta quy ước nếu $i < j $ thì tổng sẽ tăng). Tuy nhiên vì $a_i $ ở liền trước $b_j $, nên trước $a_i $ phải có ít nhất $j - 1 $ số, dẫn đến $i \geq j - 1 $, đồng thời $i = j - 1 $ chỉ khi ta có cấu hình $b_k = k $ với mọi $k < j $, đồng thời $i = 1 $, $a_1 = j $ và $b_j = j + 1 $. Điều này không xảy ra vì ta đã giả sử $a_1 < b_1 $.
[RIGHT][I][B]Nguồn: MathScope.ORG[/B][/I][/RIGHT]
 

thay đổi nội dung bởi: Mashimaru, 11-01-2012 lúc 03:17 PM
Mashimaru is offline   Trả Lời Với Trích Dẫn
Old 11-01-2012, 03:09 PM   #9
Hoanglong2011
+Thành Viên+
 
Tham gia ngày: Sep 2011
Bài gởi: 7
Thanks: 3
Thanked 0 Times in 0 Posts
Trích:
Nguyên văn bởi kirin View Post
Bài này mình chỉ làm đc bước tính số kẹo khi xếp xen kẽ còn b2 cm khá vớ vẩn gọi là nêu ra hướng giải là chính, không biết đc mấy điểm
Ng­ược lại với mình. B1 làm hơi bị ẩu .

Còn B2 thì sử dụng đơn biến: xét cách xếp 2n HS khác sao cho tồn tại 2 vị trí kề nhau cùng giới (giả sử là nam). Khi đó tồn tại 2 vị trí kề nhau khác cùng là nữ. Thay 2 bạn cùng giới kề nhau bằng 1 bạn cùng giới, thế thì số HS giảm đi 2. Nếu vẫn còn tồn tại 2 vị trí kề nhau như thế thì tiếp tục quy trình trên. Vì số n là hữu hạn suy ra sau 1 số hữu hạn bước, quá trình trên phải dừng, và thu được 1 dãy gồm 2m bạn xếp xen kẽ nhau, với 2m<2n. Rõ ràng số kẹo phát cho 2m bạn nhỏ hơn số kẹo phát cho 2n bạn, suy ra dpcm.
[RIGHT][I][B]Nguồn: MathScope.ORG[/B][/I][/RIGHT]
 

thay đổi nội dung bởi: Hoanglong2011, 11-01-2012 lúc 03:17 PM
Hoanglong2011 is offline   Trả Lời Với Trích Dẫn
Old 11-01-2012, 03:12 PM   #10
shido_soichua
Maths is my life
 
shido_soichua's Avatar
 
Tham gia ngày: Oct 2009
Đến từ: Ninh Bình
Bài gởi: 300
Thanks: 31
Thanked 132 Times in 76 Posts
Gửi tin nhắn qua Yahoo chát tới shido_soichua
Trích:
Nguyên văn bởi Hoanglong2011 View Post
Ng­ược lại với mình. B1 làm hơi bị ẩu .

Còn B2 thì sử dụng đơn biến: xét cách xếp 2n HS khác sao cho tồn tại 2 vị trí kề nhau cùng giới (giả sử là nam). Khi đó tồn tại 2 vị trí kề nhau khác cùng là nữ. Thay 2 bạn cùng giới kề nhau bằng 1 bạn cùng giới, thế thì số HS giảm đi 2. Nếu vẫn còn tồn tại 2 vị trí kề nhau như thế thì tiếp tục quy trình trên. Vì số n là hữu hạn suy ra sau 1 số hữu hạn bước, quá trình trên phải dừng, và thu được 1 dãy gồm 2m bạn xếp xen kẽ nhau, với 2m<2n. Rõ ràng số kẹo phát cho 2m bạn nhỏ hơn số kẹo phát cho 2n bạn, suy ra dpcm.
Thay như vậy thì số học sinh chỉ giảm đi 1 thôi chứ bạn ?
[RIGHT][I][B]Nguồn: MathScope.ORG[/B][/I][/RIGHT]
 
__________________
http://luongvantuy.org/forum.php
Chuyên Văn - Lương Văn Tụy
shido_soichua is offline   Trả Lời Với Trích Dẫn
Old 11-01-2012, 03:56 PM   #11
nhox12764
+Thành Viên+
 
nhox12764's Avatar
 
Tham gia ngày: Mar 2010
Đến từ: 12 Toán - Bến Tre
Bài gởi: 221
Thanks: 798
Thanked 128 Times in 64 Posts
Trích:
Nguyên văn bởi Hoanglong2011 View Post
Ng­ược lại với mình. B1 làm hơi bị ẩu .

Còn B2 thì sử dụng đơn biến: xét cách xếp 2n HS khác sao cho tồn tại 2 vị trí kề nhau cùng giới (giả sử là nam). Khi đó tồn tại 2 vị trí kề nhau khác cùng là nữ. Thay 2 bạn cùng giới kề nhau bằng 1 bạn cùng giới, thế thì số HS giảm đi 2. Nếu vẫn còn tồn tại 2 vị trí kề nhau như thế thì tiếp tục quy trình trên. Vì số n là hữu hạn suy ra sau 1 số hữu hạn bước, quá trình trên phải dừng, và thu được 1 dãy gồm 2m bạn xếp xen kẽ nhau, với 2m<2n. Rõ ràng số kẹo phát cho 2m bạn nhỏ hơn số kẹo phát cho 2n bạn, suy ra dpcm.
Số kẹo phát cho 2m bạn sau hiển nhiên ít hơn số kẹo phát cho 2n bạn đầu tiên, vậy làm thế nào bạn có đpcm?
[RIGHT][I][B]Nguồn: MathScope.ORG[/B][/I][/RIGHT]
 
nhox12764 is offline   Trả Lời Với Trích Dẫn
Old 11-01-2012, 04:37 PM   #12
mathstarofvn
+Thành Viên+
 
mathstarofvn's Avatar
 
Tham gia ngày: Nov 2009
Đến từ: A1 LQĐ_ĐN
Bài gởi: 60
Thanks: 4
Thanked 19 Times in 13 Posts
Trích:
Nguyên văn bởi shido_soichua View Post
Có ai làm bài này ko ?
Nếu các học sinh xếp xen kẽ thì số keeoj đúng bằng $\frac{1}{3}n(n^2-1) $
Nếu các học sinh không xếp xen kẽ xét 1 nhóm học sinh nam cạnh cùng với 2 học sinh nữ ở 2 đấu nhóm này.
Gọi k là số nữ ở bên trái nhóm nam
p là số nam ở bên trái bạn nữ đầu tiên bên trái nhóm nam này
p là số nam ở bên trái bạn nữ đầu tiên bên phải nhom nam này.
Khi đó ta đổi chô 2 bạn nam trên với 2 bạn nữ ở cạnh. Gọi $S_m $ là số kẹo sau m lần chuyển đổi. Khi đó ta có:
$S_{m+1}=S_m+2q-2p-4 $ mặt khác $q>p+2 $ do có ít nhất 2 nam ở giữa nên $S_{m+1}>S_m $
Do số kẹo nhận đc có giới hạn nên sau 1 số lần chuyển đổi quá trình trên dừng lại khi đó không còn 2 bạn nam đứng cạnh nhau. Ta có ĐPCM
Theo mình thì nếu xếp xen kẽ số kẹo là nhiều nhất nhưng mà để nhiều nhất phải xếp xen kẽ là sai. thử xét cách xếp là nữ nam nam nữ nữa nam.... phần sau xen kẽ, nó vẫn cho ra giá trị lớn nhất, nên bạn cm là lớn nhất phải xen kẽ chưa đúng lắm và theo cách trên 2 bạn nam có thể cạnh nhau mà vẫn lớn nhất đấy thôi
[RIGHT][I][B]Nguồn: MathScope.ORG[/B][/I][/RIGHT]
 
mathstarofvn is offline   Trả Lời Với Trích Dẫn
Old 11-01-2012, 05:00 PM   #13
shido_soichua
Maths is my life
 
shido_soichua's Avatar
 
Tham gia ngày: Oct 2009
Đến từ: Ninh Bình
Bài gởi: 300
Thanks: 31
Thanked 132 Times in 76 Posts
Gửi tin nhắn qua Yahoo chát tới shido_soichua
Trích:
Nguyên văn bởi mathstarofvn View Post
Theo mình thì nếu xếp xen kẽ số kẹo là nhiều nhất nhưng mà để nhiều nhất phải xếp xen kẽ là sai. thử xét cách xếp là nữ nam nam nữ nữa nam.... phần sau xen kẽ, nó vẫn cho ra giá trị lớn nhất, nên bạn cm là lớn nhất phải xen kẽ chưa đúng lắm và theo cách trên 2 bạn nam có thể cạnh nhau mà vẫn lớn nhất đấy thôi
Uh chỗ này sau cùng tồn tại nhiều nhất 2 nữ cạnh nhau. sau đó từ 1 trong 2 nữa này mình thực hiện đổi chỗ nam nữ theo cặp thì không làm thay đổi tổng số kẹo cuối cùng sẽ ra xen kẽ. Tuy nhiên trong bài đã quên chỗ này huhu. Hi vọng đc 4 điểm bài này
[RIGHT][I][B]Nguồn: MathScope.ORG[/B][/I][/RIGHT]
 
__________________
http://luongvantuy.org/forum.php
Chuyên Văn - Lương Văn Tụy
shido_soichua is offline   Trả Lời Với Trích Dẫn
Old 11-01-2012, 05:15 PM   #14
nghiepdu-socap
+Thành Viên+
 
nghiepdu-socap's Avatar
 
Tham gia ngày: Apr 2010
Bài gởi: 193
Thanks: 195
Thanked 129 Times in 72 Posts
Mình làm dựa vào 3 nhận xét
NX1:số nam, số nữ từ vị trí 1 đến vị trí i bất kì chênh lệch không quá 1
NX2:không có 3 nam, 3 nữ đứng cạnh nhau
NX3:có thể đổi chỗ nam nữ khi thoả mãn 2 điều trên để đưa về xen kẽ
Mình làm hơi dài dòng nên không tiện ghi lại, mọi người thông cảm
[RIGHT][I][B]Nguồn: MathScope.ORG[/B][/I][/RIGHT]
 
nghiepdu-socap is offline   Trả Lời Với Trích Dẫn
Old 11-01-2012, 07:21 PM   #15
shido_soichua
Maths is my life
 
shido_soichua's Avatar
 
Tham gia ngày: Oct 2009
Đến từ: Ninh Bình
Bài gởi: 300
Thanks: 31
Thanked 132 Times in 76 Posts
Gửi tin nhắn qua Yahoo chát tới shido_soichua
Trích:
Nguyên văn bởi nghiepdu-socap View Post
Mình làm dựa vào 3 nhận xét
NX1:số nam, số nữ từ vị trí 1 đến vị trí i bất kì chênh lệch không quá 1
NX2:không có 3 nam, 3 nữ đứng cạnh nhau
NX3:có thể đổi chỗ nam nữ khi thoả mãn 2 điều trên để đưa về xen kẽ
Mình làm hơi dài dòng nên không tiện ghi lại, mọi người thông cảm
Bạn giải thích cho mình cái nhận xét 1 cái. Mình ko hiểu lắm. Nam nữ sắp xếp bất kì mà
[RIGHT][I][B]Nguồn: MathScope.ORG[/B][/I][/RIGHT]
 
__________________
http://luongvantuy.org/forum.php
Chuyên Văn - Lương Văn Tụy
shido_soichua is offline   Trả Lời Với Trích Dẫn
Trả lời Gởi Ðề Tài Mới

Bookmarks

Ðiều Chỉnh
Xếp Bài

Quuyền Hạn Của Bạn
You may not post new threads
You may not post replies
You may not post attachments
You may not edit your posts

BB code is Mở
Smilies đang Mở
[IMG] đang Mở
HTML đang Tắt

Chuyển đến


Múi giờ GMT. Hiện tại là 03:58 PM.


Powered by: vBulletin Copyright ©2000-2024, Jelsoft Enterprises Ltd.
Inactive Reminders By mathscope.org
[page compression: 110.33 k/126.37 k (12.69%)]